akshaypatil6263 wrote:
Am unable to connect the answer to A ..... I thought its E
Official Explanation
Step 1: Identify the Question Type
The question stem asks you to “support” a particular “prediction”; this is a Strengthen the Conclusion question type.
Step 2: Deconstruct the Argument
The conclusion is located in the question stem: The Gold Standard shoe line will be profitable. The passage states only that the costs of manufacturing this shoe are exceptionally high. Profit equals revenue minus cost. If costs are exceptionally high, the only way a profit can be made is if revenue is also exceptionally high.
Step 3: State the Goal
Find an answer that makes the conclusion at least a little more likely to be valid. Keep an eye out for information about revenue, as that may be the assumption that is addressed in the correct answer.
Step 4: Work from Wrong to Right
(A) CORRECT. Strengthen. If some potential customers are willing to pay exceptionally high prices, then the exceptionally high costs might be offset enough for the shoe line to be profitable. (Note that this answer doesn't indicate that the new shoe line definitely will be profitable, only that it is a little more likely to be.)
(B) Slash. A higher sales rate than projected does not actually give you any information about profitability. In any case, the results of past releases are not necessarily indicative of the case at hand.
(C) Slash. One can argue that this is good for Brand X, in that it will mean that there is one less competitor, or that this is bad for Brand X, in that it is indicative of a sagging sneaker market. In any case, there is no direct connection between this rival brand and the potential profitability of The Gold Standard.
(D) Slash. You have been told nothing that connects the market to profitability. The size of the market does not necessarily have any bearing on profitability.
(E) Slash/Weaken. This is perhaps one reason why manufacturing costs are so high, but you already knew the costs were high from the argument. If anything, this piece of information weakens the conclusion by providing more information about the high costs.
Answer: A
Hope it helps